Prove equation in Green's function.

In summary: But I still do not understand how to show |u| and |∂h/∂n| are bounded near that point. Can you give me some hint or more detailed explanation?Sure, no problem. To show that |u| and |∂h/∂n| are bounded near (x0, y0), you can use the fact that u is continuous and h is harmonic. This means that both u and h have continuous derivatives, so they are both bounded on a small disk around (x0, y0). Therefore, their absolute values are also bounded on this disk. This means that |u| and |∂h/∂n| are both bounded near (x0,
  • #1
yungman
5,718
241

Homework Statement



Green's function [itex]G(x_0,y_0,x,y) =v(x_0,y_0,x,y) + h(x_0,y_0,x,y)[/itex] in a region [itex]\Omega \hbox { with boundary } \Gamma[/itex]. Also [itex]v(x_0,y_0,x,y) = -h(x_0,y_0,x,y)[/itex] on boundary [itex]\Gamma[/itex] and both [itex]v(x_0,y_0,x,y) \hbox { and }h(x_0,y_0,x,y)[/itex] are harmonic function in [itex]\Omega[/itex]

[tex]v=\frac{1}{2}ln[(x-x_0)^2 + (y-y_0)^2] [/tex]




Let u be continuous and h is harmonic on an open disk around [itex](x_0,y_0)[/itex] in [itex]\Omega [/itex]. Show that

[tex]_r\stackrel{lim}{\rightarrow}_0 \int_{C_r} u\frac{\partial h}{\partial n} ds = 0[/tex]

Hint from the book: Both |u| and [itex] |\frac{\partial h}{\partial n}|[/itex] are bounded near [itex](x_0,y_0)[/itex], say by M. If [itex] I_r[/itex] denotes the integral in question, then [itex] |I_r| \leq 2\pi M r \rightarrow 0 \hbox { as } r\rightarrow 0[/itex]

Homework Equations



Green's 1st identity:

[tex] \int _{\Omega} ( u\nabla^2 h + \nabla u \cdot \nabla h ) dx dy = \int_{\Gamma} u \frac{\partial h}{\partial n} ds[/tex]




The Attempt at a Solution



[tex] h=-v \hbox { on } \Gamma \Rightarrow\; \int _{\Omega} ( u\nabla^2 h + \nabla u \cdot \nabla h ) dx dy = -\int_{\Gamma} u \frac{\partial v}{\partial n} ds = -\int_{\Gamma} u \frac{1}{r} ds = -\int^{2\pi}_{0} u d\theta[/tex]

h is harmonic in [itex]\Omega\;\Rightarrow \nabla^2 h = 0[/itex]

[tex]\Rightarrow\; \int _{\Omega} \nabla u \cdot \nabla h \; dx dy = -\int^{2\pi}_{0} u d\theta[/tex]
I really don't know how to continue, please help.
 
Last edited:
Physics news on Phys.org
  • #2
h = -v on Γ. You're not integrating over Γ; you're integrating around a small circle Cr around (x0, y0). You haven't said what h is, but by the hint, presumably ∂h/∂n is bounded on some disk centered at (x0, y0). This is certainly not true if h = -v on this disk, as ∂v/∂n = 1/r is not bounded. If h has no singularity at (x0, y0), then certainly ∂h/∂n is bounded on such a disk, as h has a continuous derivative (since by assumption h is harmonic).

Apply the hint.
 
  • #3
adriank said:
h = -v on Γ. You're not integrating over Γ; you're integrating around a small circle Cr around (x0, y0). You haven't said what h is, but by the hint, presumably ∂h/∂n is bounded on some disk centered at (x0, y0). This is certainly not true if h = -v on this disk, as ∂v/∂n = 1/r is not bounded. If h has no singularity at (x0, y0), then certainly ∂h/∂n is bounded on such a disk, as h has a continuous derivative (since by assumption h is harmonic).

Apply the hint.

Thanks for you help, let me try this:

Let u be continuous and h is harmonic on an open disk around [itex](x_0,y_0)[/itex] in [itex]\Omega [/itex]. Show that

[tex]_r\stackrel{lim}{\rightarrow}_0 \int_{C_r} u\frac{\partial h}{\partial n} ds = 0[/tex]

Hint from the book: Both |u| and [itex] |\frac{\partial h}{\partial n}|[/itex] are bounded near [itex](x_0,y_0)[/itex], say by M. If [itex] I_r[/itex] denotes the integral in question, then [itex] |I_r| \leq 2\pi M r \rightarrow 0 \hbox { as } r\rightarrow 0[/itex]

So [itex]|u| \hbox { and } |\frac{\partial h}{\partial n}|\; \leq\; M[/itex]. As [itex] (x,y)\rightarrow\;(x_0,y_0)[/itex]

[tex]_r\stackrel{lim}{\rightarrow}_0 \int_{C_r} u\frac{\partial h}{\partial n} ds = \;^+_- M^2\; \int^{2\pi}_{0} r\;d\theta \;=\; _r\stackrel{lim}{\rightarrow}_0 \; (2\pi M^2r) \;=\; 0[/tex]

I have [itex] M^2 \hbox { because } u\frac{\partial h}{\partial n} \leq M^2[/itex]
 
  • #4
Yes. Now you have to prove that both |u| and |∂h/∂n| are bounded near that point.
 
  • #5
adriank said:
Yes. Now you have to prove that both |u| and |∂h/∂n| are bounded near that point.

Thanks for your time. But I got M^2 instead of M. It does not matter because r goes to zero.
 

What is a Green's function?

A Green's function is a mathematical tool used in solving differential equations. It represents the solution to a differential equation with a delta function as the forcing term. It is named after British mathematician George Green.

What is the purpose of using Green's function to prove an equation?

The use of Green's function allows for the solution of a differential equation to be written in terms of an integral, making it easier to solve and understand. It also provides a general solution that can be applied to a wide range of problems.

How is Green's function related to boundary value problems?

Green's function is often used to solve boundary value problems in physics and engineering, where the behavior of a system is dependent on its boundary conditions. It is a powerful tool for finding solutions to these types of problems.

What are some applications of Green's function?

Green's function has many applications in various fields of science and engineering. It is used in electromagnetics, fluid dynamics, quantum mechanics, and many other areas where differential equations are present.

What are the limitations of using Green's function to prove an equation?

Green's function can only be used for linear differential equations with constant coefficients. It also assumes that the boundary conditions are homogeneous, which may not always be the case in real-world problems. Additionally, the use of Green's function can be computationally intensive, making it impractical for some problems.

Similar threads

  • Calculus and Beyond Homework Help
Replies
6
Views
852
  • Calculus and Beyond Homework Help
Replies
3
Views
1K
  • Calculus and Beyond Homework Help
Replies
1
Views
705
  • Calculus and Beyond Homework Help
Replies
6
Views
1K
  • Calculus and Beyond Homework Help
Replies
3
Views
273
  • Calculus and Beyond Homework Help
Replies
2
Views
871
  • Calculus and Beyond Homework Help
Replies
5
Views
619
  • Calculus and Beyond Homework Help
Replies
10
Views
362
  • Calculus and Beyond Homework Help
Replies
20
Views
2K
  • Calculus and Beyond Homework Help
Replies
10
Views
4K
Back
Top